Find the coordinates of a point that divides the directed line segment PQ in the ratio
5:3.
A) (4,5)
B) (-6, 6)
C) (2, 2)
D) (4,1)

Answers

Answer 1

Answer:

C

Step-by-step explanation:

Let the coordinates be (m, n) then by using the section formula we have

m=(5*8+3*(-8))/(5+3)=2

n=(5*(-1)+3*(7))/(5+3)=2

The point is (2,2)


Related Questions

the table shows the ratio between the number of books ordered and their cost. 7,14 8,16 9,18. find equivalent ratios when the number of books ordered is 1,2, and 3. write them as ordered pairs in the form (x-coordinate, y-coordinate). starting from the origin explain how to plot three equivalent ratios on a coordinate grid.

Answers

Answer:

Number of Books. Cost (dollars). 7 14. 8 16. 9 18. Find equivalent ratios when the number of books ordered is 1, 2, and 3. Write them as ordered pairs in the form...

what is the 13mm = to

Answers

About 1.3 cm or it could be 1.3 cm

Triangle ABC has vertices A(0, 2), B(5, 4), and C(5,0). Triangle A'B'C'has
corresponding vertices A'(-5, -1), B'(0, 1), and C'(0, -3). Which of the following
describes a translation that would produce triangle A'B'C' from the pre-image of
triangle ABC?
A) To the left five units, up three units.
B) To the left five units, down three units.
C) To the right five units, up three units.
D) To the right five units, down three units.

Answers

Answer:

B) To the left five units, down three units

Answer:

( B ) To the left five units, down three units.

Find the coefficient of x^4 in the expansion of
(2x+1)^12

Answers

Answer:

7920

Step-by-step explanation:

you can use the binomial theorem obviously but you can also use symbolabs or wolframalpha and just solve it like that.

Please help me with this question.

Answers

I believe this question is to do with bounds so the LB (lower bound) of 56= 55.5 the LB of 9.2= 9.15
LB= 55.5/9.15= 6.06557377

Need help ASAP !!! Am I correct? Just checking….

Answers

Answer:

Yes you are

Step-by-step explanation:

If you multiply the first equation by 3 , y becomes 3y and so you can eliminate from there.

(a) 88.83 is what percent of 21?

(b) 25.83 is 52.5% of what number?


Thanks!!!

Answers

A is 4.23% of 21
B is 0.492

[tex]\sqrt{x^{2} +2x+2\\}[/tex]

Answers

Answer:

[tex]{ \tt{ \sqrt{ {x}^{2} } + 2x + 2}} \\ { \tt{ = x + 2x + 2}} \\ = { \tt{3x + 2}}[/tex]

Find the number of unique permutations of the word ALGEBRA.

Answers

Answer:

= 2520

Step-by-step explanation:

In 2520 distinct way the letter "ALGEBRA " can be Arranged

I think it's the answer

in a right triangle ABC, C is the right angle. What does cos A equal
1: cos(B)
2: both A and C
3: Cos(90-A)
4: sin(90-A)​

Answers

Answer:

4

Step-by-step explanation:

A+B+C = 180

C = 90

A+B = 90

A=90-B

Cos (A) = Sin(90-A)

Can someone help me please with this math homework!!!

Answers

Answer:

-2x + 3y = -19

Step-by-step explanation:

y + 1 = 2/3(x-8)

multiply by 3 to get rid of the fraction

3(y + 1 = 2/3(x-8)

3y + 3 = 2(x-8)

simplify

3y + 3 = 2x - 16

3y - 2x = -19

-2x + 3y = -19

Find the missing segment in the image below

Answers

Answer:

20.4

Step-by-step explanation:

that is the procedure above

Answer:

Step-by-step explanation:

Derrick's dad bought new tires for $900 using a credit card. His card has an
interest rate of 19%. How much interest will he accrue after 4 months?

Answers

Answer:

$57 interest

Step-by-step explanation:

For an interest rate of 19% ; the monthly interest will be 19/100 = 0.19

4 months interest will be (0.19 * 4)/12 = 0.0633

so now, the value of the interest will be

0.0633 * 900 = $57

A line segment has one endpoint of A (-3,2) and a slope of (-3/5). Find coordinates for B

Answers

Answer:

Step-by-step explanation:

This is a very interesting problem. We are given the slope and one point, which makes this funner than normal ;)

Filling in the slope formula with what we are given:

[tex]-\frac{3}{5}=\frac{y-2}{x-(-3)}[/tex] which simplifies to

[tex]-\frac{3}{5}=\frac{y-2}{x+3}[/tex]

Since slope is rise over run, the rise number is -3; the run number is 5. That means that the equation to find the y coordinate comes from -3 = y - 2; the equation to find the x coordinate comes from 5 = x + 3 (keep in mind that you could have made the x coordinate the negative one; it doesn't matter. You will get the same answer regardless...promise).

Solving for y:

-3 = y - 2 and

y = -1

Solving for x:

5 = x + 3 and

x = 2

Let's check those coordinates in the slope formula and see if our slope is in fact -3/5:

[tex]m=\frac{-1-2}{2-(-3)}=-\frac{3}{5}[/tex] so we're all good!

The coordinates of B are (2, -1)

FOR BRAINLIEST ASAP!!!

Jean bought an mp3 player and a calculator for a total of $208. the mp3 player cost $140 more than the calculator, what did the calculator cost

Answers

Answer:

Calculator cost = $34

Step-by-step explanation:

x + (x+140) = 208

2x + 140 = 208

x=34

solve h(x)=x^2-2x-8>7

Answers

Answer:

x < -3 and x > 5

Step-by-step explanation:

x² - 2x - 8 > 7

~Rewrite in standard form

x² - 2x - 15 > 0

~Factor

(x + 3)(x - 5) = 0

~Solve for both factors

x + 3 = 0

x = -3

x - 5 = 0

x = 5

Best of Luck!

The equation 9 y minus 6 x = 36 is written in standard form. What is the first step when writing an equivalent equation which solves for x? Divide both sides of the equation by 36. Multiply both sides of the equation by 6. Add 9y to both sides of the equation. Subtract 9y from both sides of the equation.

Answers

Answer:

D. Subtract 9y from both sides of the equation.

Step-by-step explanation:

A. Divide both sides of the equation by 36.

B. Multiply both sides of the equation by 6.

C. Add 9y to both sides of the equation.

D. Subtract 9y from both sides of the equation.

Given:

9y - 6x = 36

To solve for x

Step 1: subtract 9y from both sides

9y - 6x - 9y = 36 - 9y

- 6x = 36 - 9y

Step 2: Divide both sides by -6

- 6x / -6 = (36 - 9y) / - 6

x = (36 - 9y) / - 6

The answer is

x = (36 - 9y) / - 6

Answer:

d

Step-by-step explanation:

Find f(-2) for f(x) = 2•3^x

Answers

Answer:

2/9

Step-by-step explanation:

f(x) = 2•3^x

Let x = -2

f(-2) = 2•3^-2

We know that a^-b = 1/a^b

f(-2) = 2• 1/3^2

     = 2/9

f(x) = 2 * 3^x

f(-2) = 2 * 3^-2

f(-2) = 2 * 1/3^2

f(-2) = 2 * 1/9

f(-2) = 2/9

Hope this helps!

6.7.37
Question Help
Ay
60
a
A teenager starts a company selling personalized coffee mugs. The profit
function, in dollars, for producing and selling x mugs is
f(x) = -0.4x?
+ 12x-50, whose graph is shown.
40-
20-
0
TO
-20-
20
a. What are the start-up costs for the teenager's company?
b. How many mugs must the teenager sell before she breaks even?
c. How many mugs will give the maximum profit?
d. What will the profit be if she sells 10 mugs?
30
-407
-60-

Answers

Answer:

a. The start-up cost for the teenagers company is $50

b. The number of mugs the teenager must sell to break even are 5 mugs or 25 mugs

c. The number of mugs that will give maximum profit is 15 mugs

d. The profit if she sells 10 mugs is $30

Step-by-step explanation:

The given profit function for selling x number of mugs is presented as follows;

f(x) = -0.4·x² + 12·x - 50

a. The start-up cost in dollars is given by the value of the profit function at the start, where, x = 0, as follows;

Start-up cost = f(0) = -0.4×0² + 12×0 - 50 = -50

The negative sign represents amount put in the business

The start-up cost = (The initial) $50 put into the business.

b. The break even point is the point where, the revenue and costs are equal

At break even point; Revenue = Cost

∴ Profit, at break even point, f(x) = Revenue - Cost = 0

From the profit function, we get;

At the break even point, f(x) = 0 = -0.4·x² + 12·x - 50

Dividing by -0.4 gives;

0/(-0.4) = 0 = (-0.4·x² + 12·x - 50)/(-0.4) = x² - 30 + 125

0 = x² - 30 + 125

∴ (x - 25)·(x - 5) = 0

The number of mugs the teenager must sell before she breaks even, x = 5 mugs or x = 25 mugs.

c. From the general form of a quadratic equation,  which is; y = a··x² + b·x + c, the formula for the x-values at the maximum point is; x = -b/(2·a)

Comparing the profit function to the general form of the quadratic equation we have at the maximum point;

x = -12/(2×(-0.4)) = 15

Therefore, the number of mugs that will give maximum profit, x = 15 mugs.

d. The profit from selling 10 mugs, f(10) is given as follows;

f(10) = -0.4 × 10² + 12 × 10 - 50 = 30

The profit from selling 10 mugs, f(10) = $30

express the equation for
[tex] \frac{15y - 10}{3x} [/tex]
=6 in the form of mx+b​

Answers

Answer:

is it 6 with the 10

Step-by-step explanation:

Answer:

y = 6/5 x + 2/3

Step-by-step explanation:

A equation is given to us and we need to express it in , mx + b form . The given equation to us is,

[tex]\rm\implies 6 = \dfrac{15y - 10}{3x} [/tex]

Move 3x to LHS , we will get ,

[tex]\rm\implies 6 * 3x = 15y - 10 \\\\\rm\implies 18x = 15y - 10 \\\\\rm\implies 15y = 18x + 10 \\\\\rm\implies y =\dfrac{18x+10}{15} \\\\\rm\implies \boxed{ \bf y = \dfrac{6}{5}x + \dfrac{2}{3}}[/tex]

This is the required answer .

y=x²+4x-3 y=5x+3 solve the simultaneous equation​

Answers

Answer:

(-2, -7) or (3, 18)

Step-by-step explanation:

Start with your 2 equations

y = x^2 + 4x - 3

y = 5x + 3

Because both equal y, we substitute:

x^2 +4x -3 = 5x+ 3

x^2 - x - 6 = 0

Factor:

(x + 2) (x - 3) = 0

Solutions for x: -2 or 3 since if either of the numbers in parentheses is 0, the equation is 0 (0* anything = 0).

Then plug -2 in for x in either equation:

y = 5* -2 + 3

y = -7

So ( -2, -7) is a solution.

Plug 3 in for x in either equation:

y = 5*3 + 3

y = 18

So ( 3 , 18) is a solution.

Hope that helps, let me know if I did anything wrong!

Answer:

(- 2, - 7 ) and (3, 18 )

Step-by-step explanation:

Given the 2 equations

y = x² + 4x - 3 → (1)

y = 5x + 3 → (2)

Substitute y = x² + 4x - 3 into (2)

x² + 4x - 3 = 5x + 3 ( subtract 5x + 3 from both sides )

x² - x - 6 = 0 ← in standard form

(x - 3)(x + 2) = 0 ← in factored form

Equate each factor to zero and solve for x

x + 2 = 0 ⇒ x = - 2

x - 3 = 0 ⇒ x = 3

Substitute these values into (2) for corresponding values of y

x = - 2 : y = 5(- 2) + 3 = - 10 + 3 = - 7 ⇒ (- 2, - 7 )

x = 3 : y = 5(3) + 3 = 15 + 3 = 18 ⇒ (3, 18 )

The selling price of an apartment dropped from 250,000 to 225,000. By What percent did the price drop?

Answers

Answer:

10%

Step-by-step explanation:

The percentage decrease in price = [tex]\frac{decrease amount}{original amount}[/tex] x 100%

Original amount = 250,000

New amount = 225,000

Decrease in amount = 250,000 - 225,000

                        = 25,000

So that;

Percentage decrease in price = [tex]\frac{25000}{250000}[/tex] x 100%

                                           = 0.1 x 100%

Percentage decrease in price = 10%

The price of the apartment dropped by 10% of its original price.

Some students were asked how many pens they were carrying in their backpacks. The data is given in this frequency table. What is the mean number of pens carried by these students in their backpacks?

A. 3
B. 3.5
C. 4
D. 4.5

Answers

Answer:

4

Step-by-step explanation:

Mean: the sum of all numbers divided by the amount of numbers

In this case it will be the sum of all frequency divided by the amount of pens

Mean = 5 + 7 + 3 + 5 + 3 + 2 / 6

Mean = 25/6

Mean = 4.16

The mean is approximately 4

The answer is going to be A

The area of a circle is 80.86cm2.
Find the length of the radius rounded to 2 DP.

Answers

Answer:

Solution given:

The area of a circle =80.86cm²

we have:

The area of a circle =πr²

substituting value of area of circle we get

80.86cm=3.14*r²

dividing both side by 3.14

80.86/3.14=3.14*r²/3.14

25.75=r²

doing square root on both side

[tex]\sqrt{25.75}=\sqrt{r²}[/tex]

r=5.07cm

The length of radius is 5.07cm

The length of the radius is 5.07 cm.

What is the area of the circle?

The area of the circle is given by;

[tex]\rm Area \ of \ circle=\pi r^2\\\\Where: \ r = radius[/tex]

The area of a circle is 80.86cm^2.

The length of the radius is;

[tex]\rm Area \ of \ circle=\pi r^2\\\\ 80.86=\pi r^2\\\\r^2=\dfrac{80.86}{3.14}\\\\r^2=25.75\\\\r=5.07[/tex]

Hence, the length of the radius is 5.07 cm.

Learn more about radius here;

https://brainly.com/question/11952845

#SPJ2

how many metres are there in ½ of ⅕ km​

Answers

Step-by-step explanation:

1/5=1/5×1000=200

1/2 of 200

1/2×200=100

math math uhhh i no like math
n33d h319 pls

Answers

10. A

hope it helps✓✓✓✓✓✓✓✓✓

Answer: A for 10

Step-by-step explanation:

geometry, trigonometry

Answers

Answer:

Hello

Answer B: 32.58°

Step-by-step explanation:

[tex]sin (\widehat{A})=\dfrac{35}{65} \\\\\widehat{A}=arcsin(\dfrac{7}{13} )=32.5789...^o[/tex]

Solve for the value of x

Answers

Answer:

x = 2

Step-by-step explanation:

Given 2 intersecting chords in a circle, then

The product of the parts of one chord is equal to the product of the parts of the other chord , that is

12x = 6(x + 2) = 6x + 12 ( subtract 6x from both sides )

6x = 12 ( divide both sides by 6 )

x = 2

Answer:

[tex]x=2[/tex]

Step-by-step explanation:

The term 'secant' refers to a line segment that intersects a circle in two places. When two secants intersect inside a circle, one can use the product of lengths theory to form ratios between the parts of intersection between the secants. This ratio can be described as the following, let ([tex]secant_A[/tex]) and ([tex]secant_B[/tex]) represent the two secants in the circle. Part (1) and (2) will refer to the two parts formed after the intersection of the secants.

[tex](secant_A_1)(secant_A_2)=(secant_B_1)(secant_B_2)[/tex]

Use this formula in the given situation, substitute the given values in and solve for the unknown,

[tex](secant_A_1)(secant_A_2)=(secant_B_1)(secant_B_2)[/tex]

[tex](6)(x+2)=(12)(x)[/tex]

Simplify,

[tex](6)(x+2)=(12)(x)[/tex]

[tex]6x+12=12x[/tex]

[tex]12=6x[/tex]

[tex]x=2[/tex]

The digits 1, 7, and 8 and 2 copies of the digit 5 are all arranged to form a 5-digit integer. How many different integers can be formed?

Answers

Step-by-step explanation:

You have 5 digits.Youdonot say if duplication is allowed. I will assume not

For the first digit you have 5 choices. One of the 5 digits is gone.

You now have 4 digits to choose from. You pick one. Now you have but 3 left.

The total answer if 5*4*3*2*1 = 120

It is possible to get two solutions when solving a radical equation.
True or false

Answers

yes it is possible, although it doesn't always occur.

Your answer is: True

Other Questions
Diane Dodd raised different fruit fly populations on different food sources. She found that after about 40 generations the evolution of reproductive isolation was under way. The mechanism of evolution responsible for this was Group of answer choices Classify the polygon as regular or irregular, and concave or convex. Imagine you could live anywhere in the world based on a location's climate. What location would you choose? Use this resource and theinformation in the tutorial to help you answer the following questions about your location of choice. I need help please ASAP Ik how to do it its easy but I wanna make sure so please help what is the length of GN, given that figure LMNO is a square PLZ HELP!!!!! you good bronbtihrwithirntibibrinibn bjnj What types of evidence does joy rely on to develop his discurssion of the two sides of this issue? How does the range of evidence affect Joy's credibility as an author? A mass weighing 4 lb stretches a spring 4in. Suppose the mass is given an additional in displacement downwards and then released. Assuming no friction and no external force, the natural frequency W (measured in radians per unit time) for the system is? (Recall that the acceleration due to gravity is 32ft/sec2). a) None of the other alternatives is correct. b) W = v2 3 c)w=212 d) w = 4/6 e) w=213 f) Although the people of various religions live here, there is religious tolerance in Nepal. Justify the statement. Is -1 rational or irrational and is 3 + -1 Rational or irrational explain if u can pls 88%19. What is the solution X in this equation? 2(3x - 7) + 4(3x + 2) = 6(5x+9) + 3 what is measurement? as Nepal is rich in water resources we can produce plenty of hydroelectricity in export it into different countries for economic benefits what do you think need to be done to achieve this Juanita has always found people to take care of her. She has convinced a neighbor to do her laundry by saying she doesn't know how, and countless coworkers have finished her projects to get them done on time. Things are no different in her interpersonal relationships, except she finds that she loses friends once they feel she is taking advantage of them.A. Personality disorders often cause a person to lose his or her job.B. Personality disorders are exhibited in a wide range of contexts.C. Personality disorders are predominately found in women. Which of these would you use to connect a mouse to a computer?O RAMO ROMO USBO TOT What was the focus of the congressional investigation into the Iran-Contra affair Write letter to your friend discussing at least three areas a school excels What challenge did newly independent countries have to face as a result of unchanged colonial boundaries?The boundaries ignored the presence of mountains and rivers.The boundaries often put rival ethnic and religious groups in the same country.The boundaries put together people who had no desire for independence.The boundaries ignored ways for countries to have access to the world's oceans. do you think that the life of elderly people suffering from diabetes mellitus should be extended by giving them artificially produced insulin Is cell division a continuous process? Illustrate with an example